Analysis – Convergence of $\frac{x_1+x_2+\cdots+x_n}{n} \to a$ Implies $\frac{x_1^p+x_2^p+\cdots+x_n^p}{n^p} \to 0$ for $p > 1$

analysislimitssequences-and-series

I'm encountering a mathematical analysis problem involving limits.

Problem. Given a non-negative sequence $\{x_n\}$ satisfying $$\lim\limits_{n\to \infty}\frac{x_1+x_2+\cdots+x_n}{n}=a,$$ where $|a|<+\infty$. Given a real number $p>1$. Prove that$$\lim\limits_{n\to \infty}\frac{x_1^p+x_2^p+\cdots+x_n^p}{n^p}=0.$$

My Approach. I initially attempted to use Stolz's theorem to prove this problem. However, it became apparent that the converse of Stolz's theorem does not necessarily hold. Subsequently, I tried employing the squeeze theorem, but I couldn't find a suitable bounding inequality. I noticed that the form of this inequality is quite similar to the discrete Hardy's inequality.

(Hardy's Inequality) Given a non-negative sequence $\{x_n\}$ and a real number $p>1$, then$$\sum_{k=1}^{n}\left(\frac{x_1+x_2+\cdots+x_k}{k}\right)^p<\left(\frac{p}{p-1}\right)^p\sum_{k=1}^{n}a_k^p.$$

It is disappointing that the inequality here behaves in the opposite direction compared to the original problem. However, I have a faint intuition that the order of $\sum\limits_{k=1}^{n}a_k^p$ is around $O(n)$.

Can anyone help me? I would be very grateful.

Best Answer

The following solution is a generalization of robjohn's answer to For a positive real sequence $(a_n)$ from exponent $p=2$ to arbitrary exponents $p > 1$.

The sequence $$ \frac{x_1+x_2+\cdots+x_n}{n} $$ is convergent and therefore bounded, let $K > 0$ be an upper bound.

Also (compare Suppose that a sequence is Cesaro summable. Prove....) $$ \frac{x_n}{n} = \frac{x_1+x_2+\cdots+x_{n-1}+x_n}{n} - \frac{n-1}{n} \cdot \frac{x_1+x_2+\cdots+x_{n-1}}{n-1} $$ converges to $a - 1 \cdot a = 0$.

Given $\epsilon > 0$ there is an index $n_0$ such that $$ \frac{x_n}{n} < c := \sqrt[p-1]{\frac{\epsilon}{2K}} $$ for $n \ge n_0$. It follows that for $n \ge n_0$ $$ \frac{1}{n^p} \sum_{k=n_0}^n x_k^p = \sum_{k=n_0}^n \left( \frac{x_k}{k} \right)^p \cdot \left( \frac kn\right)^p \le c^{p-1} \sum_{k=n_0}^n \frac{x_k}{k} \cdot \frac kn \\ = c^{p-1} \frac 1n \sum_{k=n_0}^n x_k \le c^{p-1} K = \frac 12 \epsilon\, . $$ Now choose $n_1 > n_0$ so large that $$ \frac{1}{n^p} \sum_{k=1}^{n_0-1} x_k^p < \frac 12 \epsilon $$ for $n \ge n_1$. Then $$ 0 \le \frac{1}{n^p} \sum_{k=1}^n x_k^p \le \epsilon $$ for $n \ge n_1$ and that proves

$$ \lim_{n \to \infty } \frac{1}{n^p} \sum_{k=1}^n x_k^p = 0 \, . $$